Página 1 de 1

184

Publicado: 02 Feb 2010, 13:59
por Derwyd
Dan por válida la 4, pero creo que la 2 también es correcta. Cuanto mayor es el índice de refracción, menor es la energía umbral para producir luz, y por tanto mas fotones de generarán.

Re: 184

Publicado: 02 Feb 2010, 14:01
por Incógnita
Yo esta no la contesté, pero luego repasando también pensé que fuera la 2. Puede ser anulable.

Re: 184

Publicado: 02 Feb 2010, 15:55
por Derwyd
Pues va a ser que no. Acabo de encontrar una fórmula para el número de fotones emitidos y depende inversamente del cuadrado del índice de refracción.

Re: 184

Publicado: 03 Feb 2010, 23:24
por pacotem
Hola Derwyd
Con esa fórmula la respuesta correcta sería la 5? ¿Dónde encontramos bibliografía?

Re: 184

Publicado: 03 Feb 2010, 23:31
por pacotem
Hola otra vez,
He encontrado esto en el google books, el problema es que no lo puedo bajar. No lo consigo con el google books.

Física Nuclear
Burcham
pag 119

Re: 184

Publicado: 03 Feb 2010, 23:37
por Derwyd
Se puede impugnar. He subido al ftp la página del Burcham donde se ve la fórmula del número de fotones emitido y que por un lado, da por válida la afirmación de que para velocidades próximas a c no depende de la velocidad de la partícula (beta tiende a 1) y por otro que depende de 1/lambda. Por lo tanto, dos válidas, anulable.

Re: 184

Publicado: 03 Feb 2010, 23:39
por pacotem
Ok ,muchas gracias

Re: 184

Publicado: 03 Feb 2010, 23:41
por Derwyd
Ahora me ha entrado la duda. El Knoll dice que la producción de fotones depende de 1/lambda^2. A ver si encuentro algo más.

Re: 184

Publicado: 03 Feb 2010, 23:46
por noelia86
Entoncs, cual es la correcta???

Re: 184

Publicado: 04 Feb 2010, 00:07
por Derwyd
Bueno, creo que ya lo tengo claro y no hay nada que rascar. El tema es que nos piden N(lambda) por intervalo de de longitudes de onda diferencial de lambda. En este caso la dependencia es 1/lambda^2.

\(\frac{dN}{dx}=\frac{2\pi z^2 \alpha}{\lambda^2}(1-\frac{1}{\beta^2 n(\lambda)})d\lambda\)

El 1/lambda del Burcham sale de integrar en las longitudes de onda, pero no es lo que piden.

EDITO: Lo subo si queréis, pero no hay nada que impugnar.

Re: 184

Publicado: 04 Feb 2010, 09:16
por pasanvin
Según lo que he visto en el ftp, a mi me parece que la más correcta es la 2, así que voy a impugnarla

Re: 184

Publicado: 04 Feb 2010, 10:31
por Derwyd
¿La 2? Si en la expresión que viene en el Burcham que subí al ftp para el número de fotones hay un n^2 en el denominador.

Re: 184

Publicado: 04 Feb 2010, 10:45
por pasanvin
Miralo bien, el factor és

\(\left( 1-\frac{1}{\beta^2 n^2}\right)\)

con lo cual, si n augmenta, este factor disminuye, ademas he encontrado una tabla de resultados experimentales en otro libro que corrobora este resultado

Re: 184

Publicado: 04 Feb 2010, 10:51
por Derwyd
Coño tienes razón, me despiste totalmente. Voy a subir la otra referencia que comentaba. ¿Puedes hacer lo mismo con la que tu dices?

Re: 184

Publicado: 04 Feb 2010, 11:07
por pasanvin
Ahora después lo subo